Вы находитесь на странице: 1из 27

Chapter 4

NET PRESENT VALUE


FORMULA
Future value: FV=

Present value: PV=

(1+r)

Ct
(1+r)t

Effective Annual Rate: EAR=

r m
(1+ ) 1
m

Future value with EAR : FV=

Future value with EAR : PV=

FV=

rT

PV= C/r
Annuity
T

FV= C*[

(1+r) 1

PV= C*[

1( 1+r )

Growing Annuity

PV= C*[

1+g T
1+r

rg

( )

(1+EAR)T
/(1+EAR)T

PROBLEMS
1. Compute the future value of $1,000 compounded annually for
a. 10 years at 5 percent.
b. 10 years at 7 percent.
c. 20 years at 5 percent.
d. Why is the interest earned in part c not twice the amount earned in part a?
Solution:
FV=

(1+r)

10

= 1628.89

10

= 1967.15

20

= 2653.3

a. FV=

1000 x(1+0.05)

b. FV=

1000 x(1+0.07)

c. FV=

1000 x(1+0.05)

d. Interest has compounded over the interest earned during first 10 years. The
interest earned in first 10 years is 628.89, which was reinvested, which has
compounded at 1.05 times each year. Hence the interest earned in part c is
not twice the amount earned in part a, rather its more than double the
amount (2.63 times).
2. Calculate the present value of the following cash flows discounted at 10
percent.
a. $1,000 received seven years from today.
b. $2,000 received one year from today.
c. $500 received eight years from today.
Solution:

PV=

Ct
t

(1+r)

= 513.16

= 1818.18

a. PV=

1000/(1+0.10)

b. PV=

2000/(1+0.10)

c. PV=

500/(1+0.10)

= 233.25

3. Would you rather receive $1,000 today or $2,000 in 10 years if the discount rate
is 8 percent?
Solution:
In order to compare these two, we need to calculate the present value of $2000.

PV=

Ct
(1+r)t

2000
(1+.08)10

= 926.4
This value is less than $1000. Hence, $1000 should be received today instead of the
2nd offer.
4. The government has issued a bond that will pay $1,000 in 25 years. The bond
will pay no interim coupon payments. What is the present value of the bond if
the discount rate is 10 percent?
Solution:

Face value
(1+r)t

PV of bond =

1000
(1+.1)25

= 92.3
5. A firm has an estimated pension liability of $1.5 million due 27 years from
today. If the firm can invest in a risk-free security that has a stated annual
interest rate of 8 percent, how much must the firm invest today to be able to
make the $1.5 million payment?
Solution:
FV=

(1+r)

1.5 M=

27

(1+0.08)

= $187,780,23

The firm must invest $187,780,23 today.


6. You have won the Florida state lottery. Lottery officials offer you the choice of
the following alternative payouts:
Alternative 1: $10,000 one year from now.
Alternative 2: $20,000 five years from now.
Which alternative should you choose if the discount rate is:
a. 0 percent?
b. 10 percent?
c. 20 percent?
d. What discount rate makes the two alternatives equally attractive to you?
Solution:

PV=

Ct
t

(1+r)

a. 0 percent
b. 10 percent
c. 20 percent

Alternative 1
(PV of $10,000 one
year from now)
10,000
9,090.91
8,333.33

Alternative 2
(PV of $20,000 five
years from now)
20,000
12,418.43
8,037.55

Decision

Alternative 2
Alternative 2
Alternative 1

Let, at discount rate r, present value of the two alternatives will be equal.

1000
1
(1+r)

(1+r)5
=2
(1+r)1

2000
5
(1+r)

(1+r) = 1.189
R= 0.189 or 18.9%
8. Suppose you bought a bond that will pay $1,000 in 20 years. No intermediate
coupon payments will be made. If the appropriate discount rate for the bond is
8 percent,
a. what is the current price of the bond?
b. what will the price be 10 years from today?
c. what will the price be 15 years from today?
Solution:
a. Current price is equal to the present value.
PV=

1000
20
(1+.08)

= 214.55
b. Price of the bond 10 years from today:
FV10= 214.55*(1+.08)10 = 463.2
c. Price of the bond 15 years from today:
FV10= 214.55*(1+.08)15 = 680.59

9. Ann Woodhouse is considering the purchase of a house. She expects that she
will own the house for 10 years and then sell it for $5 million. What is the most
she would be willing to pay for the house if the appropriate discount rate is 12
percent?
Solution:
The maximum payment for the house today is equal to the present value of $5
million to make the investment worthwhile.

PV=

5,000,000
10
(1+.12)

= 1,609,866.18

10.You have the opportunity to make an investment that costs $900,000. If you
make this investment now, you will receive $120,000 one year from today,
$250,000 and $800,000 two and three years from today, respectively. The
appropriate discount rate for this investment is 12 percent.
a. Should you make the investment?
b. What is the net present value (NPV) of this opportunity?

c. If the discount rate is 11 percent, should you invest? Compute the NPV to
support your
answer.
Solution:

b. NPV = -900k +

120 k
(1+.12)1

250 k
(1+.12)2

800 k
(1+.12)3

=-24.13 k
a. The NPV of the investment is negative and should not be made.
c. NPV = -900k +

120 k
250 k
800 k
+
+
1
2
3
(1+.11)
(1+.11)
(1+.11)

= -4.033 k
Even at 11% discount rate, the NPV of the investment is negative, so no
investment should be made.
11. You have the opportunity to invest in a machine that will cost
$340,000. The machine will generate cash flows of $100,000 at the end
of each year and require maintenance costs of $10,000 at the beginning
of each year. If the economic life of the machine is five years and the
relevant discount rate is 10 percent, should you buy the machine? What
if the relevant discount rate is 9 percent?
Solution:

At 10%,

NPV of cash inflows =

= 379.08 k

100 k
100 k
+
1
(1+.1)
(1+.1)2

100 k
(1+.1)3

100 k
100 k
+
4
(1+.1)
(1+.1)5

NPV of cash outflows=

10 k
(1+.1)3

10 k
(1+.1) 4

340 k
(1+.1)0

10 k
10 k
10 k
+
0 +
1
(1+.1)
(1+.1)
(1+.1)2

= 381. 7 k

So, NPV of the investment = -381. 7 + 379.08 = -2.62 k


At 10% discount rate the investment provides a negative NPV. The
investment should not be made.
At 9%,

NPV of cash inflows =

100 k
5
(1+.09)

100 k
(1+.09)1

100 k
100 k
+
2
(1+.09)
(1+.09)3

100 k
+
(1+.09)4

= 388.96 k

NPV of cash outflows=

10 k
(1+.09)3

10 k
(1+.09)4

340 k
10 k
10 k
+
+
0
0
1
(1+.09)
(1+.09)
(1+.09)

10 k
+
2
(1+.09)

= 382. 4 k

So, NPV of the investment = -382.4 + 388.96 = 6.56 k


At 9% discount rate the investment provides a positive NPV. The investment
can be made.
12.Today a firm signed a contract to sell a capital asset for $90,000. The firm will
receive payment five years from today. The asset costs $60,000 to produce.
a. If the appropriate discount rate is 10 percent, is the firm making a profit on
this item?
b. At what appropriate discount rate will the firm break even?
Solution:

a. NPV= -60,000 +

90,000
(1.1)5

=-41117.08

The firm is not making profit on this item.


b. In order to break-even, NPV should be equal to 0.
NPV= 0 = -60,000 +

90,000
(1+r)5

r= 0.0845 or, 8.45%


15.Suppose you deposit $1,000 in an account at the end of each of the next four
years. If the account earns 12 percent, how much will be in the account at the
end of seven years?
Solution:

Explanation

FV=

(1+r)
The deposit at the end of year
compounded over 6 years.
The deposit at the end of year
compounded over 5 years.
The deposit at the end of year
compounded over 4 years.
The deposit at the end of year
compounded over 3 years.
Total

1 will be

1000* (1.12)6

1973.82

2 will be

1000* (1.12)5

1762.34

3 will be

1000* (1.12)4

1573.52

4 will be

1000* (1.12)3

1404.93

Sum of all the


future values

6714.61

16.What is the future value three years hence of $1,000 invested in an account
with a stated annual interest rate of 8 percent,
a. compounded annually?
b. compounded semiannually?
c. compounded monthly?
d. compounded continuously?

e. Why does the future value increase as the compounding period shortens?
Solution:

r m
(1+ ) 1
m

EAR=

Future value
formula

a.
compounde
d annually
b.
compounde
d
semiannual
ly
c.
compounde
d monthly
d.
compounde
d
continuousl
y

FV=

EAR=

Calculation

FV

8%

1000*(1.08)3

1259.7

0.0816

1000*(1.081
6)3

1265.32

0.0824

1000*(1.082
4)3

1268.19

N/A

1000* e0.08*3

1271.25

r m
(1+ ) 1
m

(1+r)
FV=

(1+EAR)T

FV=

(1+EAR)T

FV=

rT

e. The shorter the compounding period, the more frequently interest is earned and
for more periods, the investment is compounded.
18.Calculate the present value of $5,000 in 12 years at a stated annual interest
rate of 10 percent, compounded quarterly.
Solution:
PV=

/(1+EAR)T

r
1
m

( )
1+

EAR =

= (1+ .1/4)4-1
=0.1038
PV=

/(1+EAR)T

= 5000/ (1.1038)12
=1528.57
19.Bank America offers a stated annual interest rate of 4.1 percent, compounded
quarterly, while Bank USA offers a stated annual interest rate of 4.05 percent,
compounded monthly.
In which bank should you deposit your money?
Solution:
Stated Annual Interest
Rate (SAIR)
Compounded
EAR=

r m
(1+ ) 1
m

Bank America
4.1%

Bank USA
4.05%

Quarterly
4.16%

Monthly
4.13%

We should deposit our money in Bank America which offers higher effective annual
rate.
20.What is the price of a British CONSOL that pays $120 annually if the next
payment occurs one year from today? The market interest rate is 15%.
Solution:
PV= C/r
=120/0.15 = 800

21.Assuming an interest rate of 10 percent, calculate the present value of the


following streams of yearly payments:
a. $1,000 per year forever, with the first payment one year from today.
b. $500 per year forever, with the first payment two years from today.
c. $2,420 per year forever, with the first payment three years from today.

Solution:
PV= C/r
Explanation
a. $1000: first
payment one
year from
today
b. $500: first
payment two
years from
today
c. $2420: first
payment three
years from
today

Calculation

1000/0 .1
By applying the PV formula for
perpetuity, we get the present
value at the end of year 1, to get
the value at year 0, we need to
discount in by (1+r)
By applying the PV formula for
perpetuity, we get the present
value at the end of year 2, to get
the value at year 0, we need to
discount in by (1+r)2

Present
value
10,000

(500/0.1)
(1+.1)

4545.45

1+0.1

(2420/0.1)

20000

22.Given an interest rate of 10 percent per year, what is the value at date t= 5
(i.e., the end of year 5) of a perpetual stream of $120 annual payments starting
at date t= 9?
Solution:

PV= C/r
By applying this formula we get value of the stream at t=8.
Value at Year 8= 120/ 0.1 =1200
To get value at t=5, we need to discount it back to 3 years: 1200/ (1.1) 3= 901.6
23. Harris, Inc., paid a $3 dividend yesterday. If the firm raises its dividend at 5
percent every year and the appropriate discount rate is 12 percent, what is the
price of Harris stock?
Solution:
Next divided= 3*(1.05)= $3.15

PV= C/(r-g)
Price of the stock= $3.15/ (0.12-0.05) = $45
26.IDEC Pharmaceuticals is considering a drug project that costs $100,000 today
and is expected to generate end-of-year annual cash flow of $50,000 forever. At
what discount rate would IDEC be indifferent between accepting or rejecting the
project?
Solution:
IDEC will be indifferent when,
NPV =0= -100,000 + 50,000/r
r= 50,000/100,000
r= 0.5 or 50%
29.Should you buy an asset that will generate income of $1,200 per year for eight
years? The price of the asset is $6,200 and the annual interest rate is 10
percent.
Solution:
T

PV= C*[

1( 1+r )

r
8

= 1200*[

1( 1+0.1 )

0.1

= 6401.9, which is higher than 6200. So the project will generate a positive NPV
(201.9).
Decision: Buy the asset.
30.What is the present value of end-of-year cash flows of $2,000 per year, with the
first cash flow received three years from today and the last one 22 years from
today? Use a discount rate of 8 percent.
Solution:
Cash flows are received for 19 years.
T

PV= C*[

1( 1+r )

19

1( 1+0.08 )
0.08

Value at the end of year 2 =2000 *[

=19,207.2

We need to discount it for 2 years to get the present value,


PV = 119,207.2/ (1.08)2 = 16,467.1
31.What is the value of a 15-year annuity that pays $500 a year? The annuitys
first payment is at the end of year 6 and the annual interest rate is 12 percent
for years 1 through 5 and 15 percent thereafter.
Solution:

15%
12%
0

15

Cash flows are received for 15 years.


T

PV= C*[

1( 1+r )

r
15

1( 1+0.15 )
0.15

Value at the end of year 2 =500 *[

=2923.7

We need to discount it for 5 years at the discount rate of 12% to get the present
value,
PV = 2923.7/ (1.12)5 = 1659
32.You are offered the opportunity to buy a note for $12,800. The note is certain to
pay $2,000 at the end of each of the next 10 years. If you buy the note, what
rate of interest will you receive?
Solution:
10

NPV= 0 = -12,800+ 2000* [

1( 1+r )
r

10

12,800/2,000 = 6.4=

1( 1+r )
r

From, annuity factor table, r= 0.09 or 9%

33.You need $25,000 five years from now. You budget to make equal payments at
the end of every year into an account that pays an annual interest rate of 7
percent.
a. What are your annual payments?
b. Your rich uncle died and left you $20,000. How much of it must you put into
the same account as a lump sum today to meet your goal?
Solution:
T

a. FV= C*[

(1+r) 1

r
5

25000= C* [

(1+.07) 1

0.07

C= 4347.27
b. The lump sum amount must be equal to the present value of 25000, which is
25000/ (1.07)5= 17,824.7.
34.Nancy Ferris bought a building for $120,000. She paid 15 percent down and
agreed to pay the balance in 20 equal annual installments. What are the equal
installments if the annual interest rate is 10 percent?
Solution:
Down payment= 0.15* $120,000= 18,000
Balance= 102,000
20

102,000= C*[

1( 1+0.1 )
0.1

C= $11,980.9
35.You have recently won the super jackpot in the Illinois state lottery. On reading
the fine print, you discover that you have the following two options:
a. You receive $160,000 at the beginning of each year for 31 years. The income
would be taxed at a rate of 28 percent. Taxes are withheld when the checks are
issued.
b. You receive $1,750,000 now, but you do not have access to the full amount
immediately. The $1,750,000 would be taxed at 28 percent. You are able to take
$446,000 of the after-tax amount now. The remaining $814,000 will be placed in

a 30- year annuity account that pays $101,055 on a before-tax basis at the end
of each year.
Using a discount rate of 10 percent, which option should you select?
Solution:
T

1( 1+r )

PV= C*[

a. After tax payment at the beginning of each year= 160,000*(1-.28)= 115,200


30

1( 1+0.1 )
0.1

PV= 115,200+ 115,200* [

= 1,201,180.55

b. After tax payment (immediate)= 446,000


After tax payment each year= 101,055*(1-.28)= 72,759.6
30

PV= 72,759.6* [

1( 1+0.1 )
0.1

=685,898.5
Present value of total payment= 446,000 + 685,898.5 = 1,131,897.5
The present value of option a is higher, so that one should be chosen.
36.You are saving for the college education of your two children. They are two
years apart in age; one will begin college in 15 years, the other will begin in 17
years. You estimate your childrens college expenses to be $21,000 per year per
child. The annual interest rate is 15 percent. How much money must you
deposit in an account each year to fund your childrens education? You will
begin payments one year from today. You will make your last deposit when your
oldest child enters college.
Solution:
T

PV= C*[

1( 1+r )

15%
0

15

17

PV = 21000 *[

1(1+0.15)

0.15

= 59,954.55
This is the beginning value of the childrens college expenses. For the 1 st child this is
the PV 14 years from now, and for the 2nd one 16 years from now.
First child,
PV of 59,954.55= 59,954.55/(1.15)14 = 8473.3
2nd child,
PV of 59,954.55= 59,954.55/(1.15)16 = 6407.03
Total present value = 8473.3 + 6407.03= 14,880.33
Total number of deposits= 15
To find the deposit amount each year,
15

PV = 14,880.33= C*[

1(1+0.15)
0.15

C= 2544.8
37.A well-known insurance company offers a policy known as the Estate Creator
Six Pay. Typically the policy is bought by a parent or grandparent for a child at
the childs birth. The details of the policy are as follows: The purchaser (say, the
parent) makes the following six payments to the insurance company.
First birthday $750; Fourth birthday $800
Second birthday $750 ; Fifth birthday $800
Third birthday $750 ; Sixth birthday $800
No more payments are made after the childs sixth birthday. When the child
reaches age 65, he or she receives $250,000. If the relevant interest rate is 6
percent for the first six years and 7 percent for all subsequent years, is the
policy worth buying?
Solution:

-750

-750

-750

-800

-800

-800

25,00
0

birth

1st
bd

2nd
bd

3rd
bd

4th
bd

5th
bd

6th
bd

65th
bd
7%

6%

NPV of

the investment = (-750)*[

1(1+0.06)

0.06

+ {(-800)*[

1(1+0.06)

0.06

}/(1.06)3 + 25000/(1.07)64

= -750* 2.673 (800* 2.673)/(1.06)3 + 25000/(1.07)59 + [250,000/(1.07)59]/


(1.06)6
= -2004.75 -1795.44 + 3254.33
= -2545.86
The negative NPV suggests that the policy is not worth buying.
38.Your company is considering leasing a $120,000 piece of equipment for the next
10 years. Your company can buy the equipment outright or lease it. The annual
lease payments of $15,000 are due at the beginning of each year. The lease
includes an option for your company to buy the equipment for $25,000 at the
end of the leasing period (i.e., 10 years). Should your company accept the lease
offer if the appropriate discount rate is 8 percent a year?
Solution:
The first payment will be at t= 0.
9

PV of annual lease payment= 15000+ 15000*[

1(1+0.08)

0.08

= 15000 + 15000*6.2469
= 108,703.5
PV of equipment price = 25000/ (1.08)10 =11,579.84
Total present value of lease payments = 108,703.5 + 11,579.84 =120,283.34
This value is higher than the present buying price of the equipment.
Therefore, buying the equipment now would be a better option.
39.You are saving for your retirement. You have decided that one year from today
you will deposit 2 percent of your annual salary in an account which will earn 8
percent per year. Your salary last year was $50,000, and it will increase at 4
percent per year throughout your career. How much money will you have for
your retirement, which will begin in 40 years?
Solution:

Current salary = $50,000


2% of which is $50,000*0.02= 1000
This amount will grow by 4% per year for next 40 years. The amount =1000*1.04=
1040
The formula for growing annuity will be applied here.
T

1+g
1
PV= C*[
1+r

rg

( )

40

1+.04
1
= 1040*[
1+.08

0.080.04

=$20,254.18
Future value of this amount at t=40, is the amount that Ill have for my retirement.
$20,254.18* (1.08)40 = $440,012.37
40.You must decide whether or not to purchase new capital equipment. The cost of
the machine is $5,000. It will produce the following cash flows. The appropriate
discount rate is 10 percent.
Year
Cash
Flow
1
$ 700
2
900
3
1,000
4
1,000
5
1,000
6
1,000
7
1,250
8
1,375
Solution:
Year
0
1

Cash
Flow ($)
-5000
700

900

1,000

1,000

Discount
Present
factor
value
1
-5000
1/
636.36
(1+0.1)1
1/
743.8
2
(1+0.1)
1/
751.31
3
(1+0.1)
1/
683.01

1,000

1,000

1,250

1,375

(1+0.1)4
1/
(1+0.1)5
1/
(1+0.1)6
1/
(1+0.1)7
1/
(1+0.1)8
NPV

620.92
564.47
641.45
641.45
282.77

Decision: Positive NPV suggests to purchase the equipment.


41.Your younger brother has come to you for advice. He is about to enter college
and has two options open to him. His first option is to study engineering. If he
does this, his undergraduate degree would cost him $12,000 a year for four
years. Having obtained this, he would need to gain two years of practical
experience: in the first year he would earn $20,000, in the second year he
would earn $25,000. He would then need to obtain his masters degree, which
will cost $15,000 a year for two years. After that he will be fully qualified and
can earn $40,000 per year for 25 years.
His other alternative is to study accounting. If he does this, he would pay
$13,000 a year for four years and then he would earn $31,000 per year for 30
years.
The effort involved in the two careers is the same, so he is only interested in the
earnings the jobs provide. All earnings and costs are paid at the end of the year.
What advice would you give him if the market interest rate is 5 percent? A day
later he comes back and says he took your advice, but in fact, the market
interest rate was 6 percent. Has your brother made the right choice?
Solution:

Formula
Present
value of
undergradua
te cost

Engineering

PV= C*[

-12000* [
T

1( 1+r )

1( 1+.05 )

0.05

= -42,552

Accounting
-13000* [
4

1( 1+.05 )

0.05
= -46,098

Present
value of two
years

PV=C/(1+r)t

20,000/(1.05)5+ 25,000/
(1.05)6
=34,325.91

earnings
Present
value of
Masters cost

PV=

{-15000* [

T2

Present
value of
earning of 25
years

{C*[

1( 1+r )
2
}/(1+r)1( 1+.05 )

r
0.05

}/

(1.05)6
= -20,812.694

PV=

{C*[

{40000* [

{31000* [

1( 1+r )
25
}/(1+r)1( 1+.05 )

r
0.05
T2

}/

30

1( 1+.05 )
0.05

(1.05)8
= 381,572.25

}/

352,533.466

345,958.8076

(1.05)4
= 392,056.8076

Option 1 provides higher NPV so studying Engineering would be the suggestion.


The same procedure for 6% and then compare the NPVs.
42.Ms. Adams has received a job offer from a large investment bank as an
assistant to the vice president. Her base salary will be $35,000. She will receive
her first annual salary payment one year from the day she begins to work. In
addition, she will get an immediate $10,000 bonus for joining the company. Her
salary will grow at 4 percent each year. Each year she will receive a bonus equal
to 10 percent of her salary. Ms. Adams is expected to work for 25 years. What is
the present value of the offer if the discount rate is 12 percent?
Solution:

PV= C*[

1+g T
1+r

rg

( )

g=4%
10000 3500
0

r=10%
25

35000

As salary will increase by 4% each year, so will the bonus.


Salary at year 1= 35000
Bonus = 35000*0.1 = 3500
Total payment = (35000+ 3500)= 38500

PV of the offer = 10,000 + 38500* [

25

1+.04
= 415,783.6
1+.12

0.120.04

43.Southern California Publishing Company is trying to decide whether or not to


revise its popular textbook, Financial Psychoanalysis Made Simple. They have
estimated that the revision will cost $40,000. Cash flows from increased sales
will be $10,000 the first year.
These cash flows will increase by 7 percent per year. The book will go out of
print five years from now. Assume the initial cost is paid now and all revenues
are received at the end of each year. If the company requires a 10 percent
return for such an investment, should it undertake the revision?
Solution:

1+.07 5
1
NPV= -40,000 + 10,000* [
= 3,041.91
1+.1

0.10.07

44.Ernie Els wants to save money to meet two objectives. First, he would like to be
able to retire 30 years from now with a retirement income of $300,000 per year
for 20 years beginning at the end of the 31 years from now. Second, he would
like to purchase a cabin in the mountains 10 years from now at an estimated
cost of $350,000. He can afford to save only $40,000 per year for the first 10
years. He expects to earn 7 percent per year from investments. Assuming he
saves the same amount each year, what must Ernie save annually from years
11 to 30 to meet his objectives?
Solution:
Present value of retirement income of $300,000 for 20 years:
T

PV20= C*[

1( 1+r )

r
20

=300,000* [

1( 1+.07 )
0.07

=300,000* 10.594
= 3,178,200
This needs to be discounted again to get the present value today,
PV= 3,178,200/ (1.07)30
=417,510.97

Present value of price of cabin:


PV= 350,000/(1.07)10 = 177,922.25
Present value of saving of first 10 years:
10

PV= 40,000*[

1( 1+.07 )
0.07

= 280,943.26

Let, he must save amount C from year 11 to year 30 and he replenishes all his
savings after the 50 years.
PV =

{C* [

20

1( 1+.07 )
0.07

}/

(1.07)10 = 5.38545C

The PV of savings and PV of withdrawals (retirement and cabin) should be equal, in


other words NPV must be equal to zero.
(280,943.26+ 5.385C) =417,510.97+177,922.25
5.38545C = 314,489.96
C= 58,396.23
Yearly Saving amount= $58,396.23
45.Jack Ferguson has signed a three-year contract to work for a computer software
company. He expects to receive a base salary of $5,000 a month and a bonus of
$10,000 at year-end. All payments are made at the end of periods. What is the
present value of the contract if the stated annual interest rate, compounded
monthly, is 12 percent?
46.Peter Green bought a $15,000 Honda Civic with 20 percent down and financed
the rest with a four-year loan at 8 percent stated annual interest rate,
compounded monthly. What is his monthly payment if he starts the payment
one month after the purchase?
47.On September 1, 1998, Susan Chao bought a motorcycle for $10,000. She paid
$1,000 down and financed the balance with a five-year loan at a stated annual
interest rate of 9.6 percent, compounded monthly. She started the monthly
payment exactly one month after the purchase, i.e., October, 1998. In the
middle of October, 2000, she got a new job and decided to pay off the loan. If
the bank charges her 1 percent prepayment penalty based on the loan balance,
how much should she pay the bank on November 1, 2000?
48.When Marilyn Monroe died, ex-husband Joe DiMaggio vowed to place fresh
flowers on her grave every Sunday as long as he lived. A bunch of fresh flowers
that the former baseball player thought appropriate for the star cost about $5
when she died in 1962.
Based on actuarial tables, Joltin Joe could expect to live for 30 years after the
actress died. Assume that the stated annual interest rate, compounded weekly,
is 10.4 percent.

Also, assume that the rate of inflation is 3.9 percent per year, when expressed
as a stated annual inflation rate, compounded weekly. Assuming that each year
has exactly 52 weeks, what is the present value of this commitment?
Solution:
Total weeks in 30 years= 30*52= 1560

EAR=

0.104 52
(1+
) 1
52

= 0.1095

Effective annual inflation rate=

0.039 52
(1+
) 1
52

= 0.04

1+g T
1
PV= C*[
1+r

rg

( )

=5*[

1+.04
1+.1095
.1095.04

30

=56.61

49.In January 1984, Richard Goose Gossage signed a contract to play for the San
Diego Padres that guaranteed him a minimum of $9,955,000. The guaranteed
payments were $875,000 for 1984, $650,000 for 1985, $800,000 in 1986, $1
million in 1987, $1 million in 1988, and $300,000 in 1989. In addition, the
contract called for $5,330,000 in deferred money payable at the rate of
$240,000 per year from 1990 through 2006 and then $125,000 a year from
2007 through 2016. If the effective annual rate of interest is 9 percent and all
payments are made on July 1 of each year, what would the present value of
these guaranteed payments be on January 1, 1984? Assume an interest rate of
4.4 percent per six months. If he were to receive an equal annual salary at the
end of each of the five years from 1984 through 1988, what would his
equivalent annual salary be? Ignore taxes throughout this problem.
Solution:
Year
1 Jul 1984
1985
1986

Cash Flow ($)


875,000
650,000
800,000

Discount factor
Present value
1/(1+0.044)0
875,000
1/(1+0. 09)1
596,330.28
1/(1+0. 09)2
673,344

1987
1988
1989
1990- 2006

1/(1+0. 09)3
1/(1+0. 09)4
1/(1+0. 09)5

1,000,000
1,000,000
300,000
240,000

17

1( 1+.09 )
0.09
2007-2016

125,000

/(1.09)5
120,477.65

10

1( 1+.09 )
0.09

772,183.48
708,425.2
194,979.42
1,332,672

22

/(1.09)

Value on 1 Jul 1984

5,273,412

We need to discount in back to 1 January 1984 using the 4.4% semi-annual rate:
PV= 5,273,412/(1.044) = 5,051,160.95
50.Mike Bayles has just arranged to purchase a $400,000 vacation home in the
Bahamas with a 20% down payment. The mortgage has an 8% annual
percentage rate (APR) and calls for equal monthly payments over the next 30
years. His first payment will be due one month from now. However, the
mortgage has an 8-year balloon payment, meaning that the loan must be paid
off then. There were no other transaction costs or finance charges. How big will
Justins balloon payment be in 8 years?
Solution:
Down payment= 400,000* 20%= $80,000
m

EAR=

r
1+
1
m

( )
(1+

.08 12
) 1
12

=0.083 or 8.3%
PV of equal monthly payments= 400,000-80,000= 320,000
T

PV= C*[

1( 1+ EAR )

EAR
3012

320,000= C*[
C= 26,560

1( 1+.083 )
0.083

The balloon payment will be equal to the value of payments of last (30-8)=22 years
at the end of year 8.
2212

1( 1+.083 )
0.083

PV= 26560*[

=320,000, this is amount of the balloon payment.


51.You want to lease a set of golf clubs from Pings Ltd. for $4,000. The lease
contract is in the form of 24 months of equal payments at a 12% annual
percentage rate (APR). Suppose payments are due in the beginning of the month
and your first payment is due immediately. What will your monthly lease
payment be?
Solution:
Monthly annual rate= 12%/12= 1%
23

PV= 4000 = C+ C*[

1( 1+.01 )
0.01

C= 185.43 (thats the monthly lease payment)


52.A 10-year annuity pays $900 per year, with payments made at the end of each
year. The first $900 will be paid 5 years from now. If the APR is 8% and interest is
compounded quarterly, what is the present value of this annuity?
Solution:
EAR=

r m
(1+ ) 1
m

= (1+.08/4)4-1
=0.0824
10

PV4= 900*[

1( 1+.0824 )
0.0824

=5,9575.71

PV= 5,9575.71/(1.0824)4 = 4,353.5


53.Paul Adams owns a health club in downtown LA. He charges his customers an
annual fee of $400 and has an existing customer base of 500. Paul plans to raise
the annual fee by 10 percent every year and expects the club membership to
grow at a constant rate of 3 percent for the next five years. The overall expenses
of running the health club are $80,000 a year and are expected to grow at the
inflation rate of 2 percent annually. After five years, Paul plans to buy a luxury
boat for $500,000, close the health club, and travel the world in his boat for the
rest of his life. What is the annual amount that Paul can spend while on his world

tour if he will have no money left in the bank when he dies? Assume Paul has a
remaining life of 15 years and earns 6 percent on his saving.
Solution:
Earnings

Annual
expense
Net income

Current
$400*500=
$200,000
$80,000
$120,000

Year 1
(400* 1.1)*(
500*1.03)=
226,600
80,000*1.0
2= 81,600
145000

Growth rate of net income = (145000-12000)/12000 =.2083 or 20.83%


Compounding
Future value at
factor*
year 5
Year 1
145000
(1.08)^4
197,270.9
Year 2
145000*1.2083=
(1.08)^3
220705.95
175,203.5
Year 3
175,203.5*1.2083=
(1.08)^2
246925
211,698.4
Year 4
211,698.4*1.2083=255,7
(1.08)^1
276258.77
95.16
Year 5
255,795.16*1.2083=309, (1.08)^0
309,077.3
077.3
FV of net income from club at end of year 5
2,223,321.4
Income form year 1 will earn interest for 4 years, income from year 2 for 3 years
and so on.
Value of boat at year 5= 500,000
Remaining money= 2,223,321.4-500000= 473,083.48
Let, the annual amount that Paul can spend while on his world tour= C,
15

PV5 of which is = C*[

1( 1+.08 )
0.08

= 473,083.48

C= 55,270.13
Current earnings= 400*500= 200,000
Earnings next year= (400*1.1)*(500*1.03)= 226,600
Accumulated growth rate of earnings= (226,600-200,000)/ 200,000 =0.133
Current expense= 80,000
Expense next year = 80,000*1.02 = 81,600

Expenses
(Previous
years
expenses*1.0
2)
81,600

Net Income
available
for saving

Compoundin
g factor*

Future
value
after 5
years

Year 1

Earnings
(Previous
years
earnings*1.13
3)
226,600

145000

(1.08)^4

197,270.9

Year 2

256,737.8

83,232

173505.

(1.08)^3

218,566.3

Year 3

290,883.93

84,896.64

205987.29

(1.08)^2

240,263.6

Year 4

329,571.49

86,594.573

242976.92

(1.08)^1

262,415.1

Year 5

373,404.498

88,326.46

285087.038

(1.08)^0

FV of net income from club at end of year 5

285,087.0
38
1,203,603

*Income form year 1 will earn interest for 4 years, income from year 2 for 3 years
and so on.
Value of boat at year 5= 500,000
Remaining money= 1,203,603-500,000= 703,603
Let, the annual amount that Paul can spend while on his world tour= C,
15

PV5 of which is = C*[


C= 82,201.62

1( 1+.08 )
0.08

= 703,603

Вам также может понравиться